Diễn Đàn MathScopeDiễn Đàn MathScope
  Diễn Đàn MathScope
Ghi Danh Hỏi/Ðáp Community Lịch

Go Back   Diễn Đàn MathScope > Sơ Cấp > Việt Nam và IMO > 2012

News & Announcements

Ngoài một số quy định đã được nêu trong phần Quy định của Ghi Danh , mọi người tranh thủ bỏ ra 5 phút để đọc thêm một số Quy định sau để khỏi bị treo nick ở MathScope nhé !

* Nội quy MathScope.Org

* Một số quy định chung !

* Quy định về việc viết bài trong diễn đàn MathScope

* Nếu bạn muốn gia nhập đội ngũ BQT thì vui lòng tham gia tại đây

* Những câu hỏi thường gặp

* Về việc viết bài trong Box Đại học và Sau đại học


Trả lời Gởi Ðề Tài Mới
 
Ðiều Chỉnh Xếp Bài
Old 06-03-2012, 09:31 PM   #76
huynhcongbang
Administrator

 
huynhcongbang's Avatar
 
Tham gia ngày: Feb 2009
Đến từ: Ho Chi Minh City
Bài gởi: 2,413
Thanks: 2,165
Thanked 4,188 Times in 1,381 Posts
Gửi tin nhắn qua Yahoo chát tới huynhcongbang
Trích:
Nguyên văn bởi Win-DungDan View Post
Tôi thấy có bài BDT rất khó của bạn Cẩn trên mathlinks xin đưa cùng các bạn trải nghiệm:
Bài 12: Cho các số thực dương sao cho a+b+c = 1 . Chứng minh rằng:
$\[\frac{36}{{{a}^{2}}b\text{+}{{b}^{2}}c\text{+}{{c} ^{2}}a}\text{+}\frac{1}{abc}\ge 343\] $ .
Một bạn ở KHTN đã có lời giải cho bài toán này. Tuy không đẹp lắm nhưng chắc là dễ theo dõi hơn lời giải rất dài của tác giả.

http://www.artofproblemsolving.com/F...56953#p2256953
[RIGHT][I][B]Nguồn: MathScope.ORG[/B][/I][/RIGHT]
 
__________________
Sự im lặng của bầy mèo
huynhcongbang is offline   Trả Lời Với Trích Dẫn
The Following 2 Users Say Thank You to huynhcongbang For This Useful Post:
than-dong (07-03-2012), Win-DungDan (11-03-2012)
Old 06-03-2012, 09:32 PM   #77
avip
+Thành Viên+
 
Tham gia ngày: Sep 2010
Bài gởi: 392
Thanks: 135
Thanked 247 Times in 159 Posts
Trích:
Nguyên văn bởi Mashimaru View Post
Bài 18. Khánh và Châu chơi trò bốc sỏi từ một đống sỏi lúc đầu có $n $ viên. Hai bạn thay phiên nhau bốc sỏi sao cho số lượng sỏi bốc mỗi lượt phải có dạng $p - 1 $ với $p $ là một số nguyên tố. Ai lấy được viên sỏi cuối cùng là thắng cuộc. Khánh đi trước và hai bạn đều chơi tối ưu. Chứng minh rằng tồn tại vô hạn $n $ để Châu có chiến thuật thắng.
Trích:
Nguyên văn bởi avip View Post
Trước tiên đặt $T,S $ lần lượt là tập các số tự nhiên $n $ sao cho nếu ban đầu đống sỏi có $n $ viên thì Khánh, Châu có chiến thuật thắng. Qui ước $0 \in S $.
Ta cmđ: $S \cup T $ là một phân hoạch của $\mathbb{N} $ và $3,8 $ là hai số nguyên dương đầu tiên thuộc $S $.
Với $m \in \mathbb{N} $, đặt $S_m = \{n \in S \; | \; n < m \} $.

Giả sử ban đầu đống sỏi có $N $ viên. Ta có nhận xét: $N \in S \Leftrightarrow N - n + 1 $ không là số nguyên tố với mọi $n \in S_N \; (1) $. Thực vậy:
Chiều thuận: Giả sử $N \in S $. Ta có: dù Khánh bốc $p-1 $ viên sỏi với $p $ là số nguyên tố nào đi chăng nữa thì Châu cũng có chiến thuật thắng, tức là $N-(p-1) $ không thuộc $S $. Ta cm được chiều thuận.
Chiều đảo: tương tự.

Từ đây, giả sử $S $ hữu hạn, ta đặt $S = \{a_1, ... , a_s \} $ (tăng dần). Xét số sỏi là $N = \prod_{i=1}^s a_i -1 $. Từ mệnh đề $(1) $ dễ thấy $N \in S $ và $N > a_s $ (mâu thuẫn). Vậy ta có đpcm.
Bằng máy tính, em dự đoán một kết quả cho bài toán này (đúng đến $S_{7000} $). Mọi người cùng thảo luận xem sao.

Chứng minh $\forall n \in S $: nếu $n-3 \notin S $ thì $n+3 \in S $.
[RIGHT][I][B]Nguồn: MathScope.ORG[/B][/I][/RIGHT]
 
__________________
VIẾT CÁI CHỮ KÍ ĐỂ KHI EDIT BÀI ĐỠ XẤU
avip is offline   Trả Lời Với Trích Dẫn
Old 07-03-2012, 07:22 PM   #78
quykhtn
+Thành Viên+
 
Tham gia ngày: Mar 2012
Đến từ: Cái nôi của phở
Bài gởi: 259
Thanks: 78
Thanked 697 Times in 193 Posts
Bài 27.
Cho $ a,b,c $ là các số thực dương thỏa mãn: $ abc=1 $.
Chứng minh rằng:
$ \frac{1}{a^8+b^3+c}+\frac{1}{b^8+c^3+a}+\frac{1}{c ^8+a^3+b} \le 1 $

Bài 28.
Cho tam giác ABC và điểm M bất kì .Chứng minh rằng:
$ \frac{MA}{BC}+\frac{MB}{CA}+\frac{MC}{AB} \ge \frac{BC+CA+AB}{MA+MB+MC} $

Mai là 8-3 rồi.Chúc các bạn nữ năm nay thi TST tốt.
[RIGHT][I][B]Nguồn: MathScope.ORG[/B][/I][/RIGHT]
 

thay đổi nội dung bởi: congbang_dhsp, 08-03-2012 lúc 12:43 PM
quykhtn is offline   Trả Lời Với Trích Dẫn
Old 07-03-2012, 09:14 PM   #79
quykhtn
+Thành Viên+
 
Tham gia ngày: Mar 2012
Đến từ: Cái nôi của phở
Bài gởi: 259
Thanks: 78
Thanked 697 Times in 193 Posts
Trích:
Nguyên văn bởi 5434 View Post
------------------

Cho M nằm ngoài tam giác và $MA,MB, MC $ đều rất lớn thì baì toán sai rồi.
Nếu thế thì bất đẳng thức hiển nhiên đúng mà.
[RIGHT][I][B]Nguồn: MathScope.ORG[/B][/I][/RIGHT]
 
quykhtn is offline   Trả Lời Với Trích Dẫn
Old 07-03-2012, 09:38 PM   #80
macdangnghi
+Thành Viên+
 
Tham gia ngày: Jan 2009
Bài gởi: 11
Thanks: 1
Thanked 15 Times in 7 Posts
Bài 29.
a) Tìm tất m nguyên dương sao cho $m^2-1 $ là ước của $3^m+5^m $.

b) Tìm m nguyên dương sao cho $5^{m}+3^{m} $ chia hết cho $m^{2}-1 $.

c) Tìm các số nguyên dương $a,b,n (a>b) $ thỏa mãn $n^{a}|a^{n}+b^{n} $.

Bài 30.
Tìm hàm số $f: \mathbb{N} \to \mathbb{N} $ thỏa mãn $f(1)>0 $ và
$f(x^{4}+5y^{4}+10z^{4})=f^{4}(x)+5f^{4}(y)+10f^{4} (z) $

[RIGHT][I][B]Nguồn: MathScope.ORG[/B][/I][/RIGHT]
 

thay đổi nội dung bởi: congbang_dhsp, 08-03-2012 lúc 12:47 PM Lý do: Chỉnh sửa Latex
macdangnghi is offline   Trả Lời Với Trích Dẫn
The Following 2 Users Say Thank You to macdangnghi For This Useful Post:
5434 (10-03-2012), MK.Duy (12-03-2012)
Old 08-03-2012, 02:22 PM   #81
macdangnghi
+Thành Viên+
 
Tham gia ngày: Jan 2009
Bài gởi: 11
Thanks: 1
Thanked 15 Times in 7 Posts
Cho các số thực dương $a_{1},a^_{2},...,a_{n} $ thỏa mãn $\prod_{k=1}^{n}a_{k}=1 $ .CMR
$\sum_{k=1}^{n}\frac{1}{a_{k}+n-1}\geqslant \sum_{k=1}^{n}
\frac{1}{S+1-a_{k}} $ trong đó $S=\sum_{k=1}^{n}a_{k} $
------------------------------
Cho các số thực $x_{1},x_{2},...,x_{n} $ thỏa mãn $\sum_{k=1}^{n}x_{k}^{2}=n(n-1) $. Tìm giá lớn nhất của biểu thức
P=$\prod_{i\neq j=1}^{n}(x_{i}-x_{j})^{2} $
------------------------------
Cho một số điểm trên đường tròn đơn vị sao cho tích khoảng cách từ một điểm bất kì thuộc đường tròn đến các điểm đã cho không vươt quá 2. Chứng minh rằng các điểm đã cho là các đỉnh của một đa giác đêu.
[RIGHT][I][B]Nguồn: MathScope.ORG[/B][/I][/RIGHT]
 

thay đổi nội dung bởi: macdangnghi, 08-03-2012 lúc 02:44 PM Lý do: Tự động gộp bài
macdangnghi is offline   Trả Lời Với Trích Dẫn
The Following User Says Thank You to macdangnghi For This Useful Post:
5434 (10-03-2012)
Old 08-03-2012, 02:46 PM   #82
quykhtn
+Thành Viên+
 
Tham gia ngày: Mar 2012
Đến từ: Cái nôi của phở
Bài gởi: 259
Thanks: 78
Thanked 697 Times in 193 Posts
Trích:
Nguyên văn bởi huynhcongbang View Post

Bài 15. (tương tự TST 1992, bài 4 - TST 2007, bài 3)

Cho tam giác ABC nhọn và đặt $x=\cos A,y=\cos B,z=\cos C $. Chứng minh rằng

$\max \left\{ \frac{x}{y}+\frac{y}{x}+2;\frac{y}{z}+\frac{z}{y}+ 2;\frac{z}{x}+\frac{x}{z}+2 \right\}\ge \frac{2}{x+y+z-1}. $
Bài này có một lời giải khác như sau:
Đặt $ BC=a,CA=b,AB=c $.Không mất tổng quát,giả sử $ a \ge b \ge c $.
Ta sẽ chứng minh:
$ \frac{x}{z}+\frac{z}{x} \ge \frac{2}{x+y+z-1}-2 $.
Sử dụng định lí hàm số cosin có:
$ \frac{x}{z}+\frac{z}{x}-2 $

$=\frac{a(b^2+c^2-a^2)}{c(a^2+b^2-c^2)}+\frac{c(a^2+b^2-c^2)}{a(b^2+c^2-a^2)}-2 $


$=\frac{(a(b^2+c^2-a^2)-c(a^2+b^2-c^2))^2}{ac(a^2+b^2-c^2)(b^2+c^2-a^2)} $


$ =\frac{(a-c)^2((a+c)^2-b^2)^2)}{ac(a^2+b^2-c^2)(b^2+c^2-a^2)} $
.

Mặt khác, $ x+y+z-1=\frac{(b+c-a)(c+a-b)(a+b-c)}{2abc} $.
Do đó:
$ \frac{2}{x+y+z-1}-4 $

$=\frac{4abc}{(a+b-c)(b+c-a)(c+a-b)}-4 $

$ =\frac{4(abc-(a+c-b)(b^2-(a-c)^2))}{(a+b-c)(b+c-a)(c+a-b)} $

$ =\frac{4((a-c)^2(a+c-b)+b(b-c)(b-a))}{(b+c-a)(c+a-b)(a+b-c)} $

$ \le \frac{4(a-c)^2}{(b+c-a)(a+b-c)} $
.
Bài toán được chứng minh nếu:
$ \frac{(a-c)^2((a+c)^2-b^2)^2}{ac(a^2+b^2-c^2)(b^2+c^2-a^2)} \ge \frac{4(a-c)^2}{(b+c-a)(a+b-c)} $

Hay là:
$ ((a+c)^2-b^2)^2(b^2-(a-c)^2) \ge 4ac(b^4-(a^2-c^2)^2) $

Chú ý rằng: $ (a+c)^2-b^2-2ac=a^2+c^2-b^2 > 0 $
Và $ ((a+c)^2-b^2)(b^2-(a-c)^2)-2(b^4-(a^2-c^2)^2) $
$=(a^2-c^2)^2+2b^2(a^2+c^2)-3b^4 $
$ \ge (b^2-c^2)^2+2b^2(b^2+c^2)-3b^4=c^4 > 0 $.
Suy ra điều phải chứng minh.
Đẳng thức xảy ra khi và chỉ khi tam giác ABC đều.

Mình xin đóng góp tiếp các bài toán sau:
Bài 31.
Cho tam giác ABC nội tiếp (O),ngoại tiếp (I).(I) tiếp xúc với BC,CA,AB lần lượt tại D , E ,F.Gọi H là giao điểm của CI với DF , K là giao điểm của BI với DE và M là giao điểm của HE với KF.Chứng minh rằng O,M,I thẳng hàng.

Bài 32.
Cho tam giác ABC có độ dài ba cạnh là $ a,b,c $.Gọi $ \ m_a , \ m_b ,\ m_c $ và $ \l_a , \l_b , \l_c $ là độ dài ba đường trung tuyến và ba đường phân giác trong .Chứng minh rằng:
$ (\ m_a+\ m_b+\ m_c )\left(\frac{1}{\l_a}+\frac{1}{l_b}+\frac{1}{\l_c} \right) \ge (a+b+c)\left(\frac{1}{a}+\frac{1}{b}+\frac{1}{c} \right). $


Bài 33.
Cho $ p $ là số nguyên tố lớn hơn 3.Chứng minh rằng $ C_{2011p^{2010}-1}^{p-1}-1 $ chia hết cho $ p^{2012}. $

Bài 34.
Cho các số thực dương $ a,b,c $ thỏa mãn: $ a^2+b^2+c^2=3 $.
Chứng minh rằng:
$ \frac{a}{b+c^3}+\frac{b}{c+a^3}+\frac{c}{a+b^3} \ge \frac{3}{2}. $

[RIGHT][I][B]Nguồn: MathScope.ORG[/B][/I][/RIGHT]
 

thay đổi nội dung bởi: quykhtn, 08-03-2012 lúc 02:58 PM
quykhtn is offline   Trả Lời Với Trích Dẫn
The Following 3 Users Say Thank You to quykhtn For This Useful Post:
huynhcongbang (12-03-2012), K56khtn (20-03-2012), n.v.thanh (09-03-2012)
Old 08-03-2012, 02:52 PM   #83
macdangnghi
+Thành Viên+
 
Tham gia ngày: Jan 2009
Bài gởi: 11
Thanks: 1
Thanked 15 Times in 7 Posts
Xác định đa thức P(x) với hệ số thực thỏa mãn $P[(x+1)^{2012}]=[P(x)]^{2012}+\sum_{k=0}^{2011}C_{n}^{k}x^{k} $ với mọi x\in \mathbb{R}
[RIGHT][I][B]Nguồn: MathScope.ORG[/B][/I][/RIGHT]
 
macdangnghi is offline   Trả Lời Với Trích Dẫn
The Following 2 Users Say Thank You to macdangnghi For This Useful Post:
5434 (10-03-2012), n.v.thanh (09-03-2012)
Old 08-03-2012, 09:38 PM   #84
kien10a1
+Thành Viên+
 
kien10a1's Avatar
 
Tham gia ngày: Feb 2011
Đến từ: Vĩnh Yên- Vĩnh Phúc
Bài gởi: 371
Thanks: 43
Thanked 263 Times in 153 Posts
Gửi tin nhắn qua Yahoo chát tới kien10a1
Bài 31: Bài này có lẽ là ghép từ 2 bài toán quen thuộc.
Trước tiên, ta có kết quả OI là đường thẳng Euler của tam giác DEF.
Ta chứng minh M, trực tâm J của DEF và I thẳng hàng là xong. Hạ các đường cao FT, ES của tam giác DEF.
Có $\widehat{HED}=\widehat{HDE}=\widehat{DFK} $ nên E,F,H,K đồng viên, suy ra $\overline{MF}.\overline{MK}=\overline{Me}.\overlin e{MH} $
Xét 2 đường tròn đường kính FK và EH:thì thấy M có cùng phương tích với cả 2, I là trực tâm DHK, J là trực tâm DEF nên cũng có I,J có cùng phương tích với 2 đường tròn trên, suy ra M,I,J thẳng hàng và có ĐPCM
[RIGHT][I][B]Nguồn: MathScope.ORG[/B][/I][/RIGHT]
 
__________________
Quay về với nơi bắt đầu
kien10a1 is offline   Trả Lời Với Trích Dẫn
The Following User Says Thank You to kien10a1 For This Useful Post:
n.v.thanh (09-03-2012)
Old 09-03-2012, 10:50 AM   #85
macdangnghi
+Thành Viên+
 
Tham gia ngày: Jan 2009
Bài gởi: 11
Thanks: 1
Thanked 15 Times in 7 Posts
Bài 35: Cho tam giác $A_{1}B_{1}C_{1} $ và các điểm A,B,C theo thứ tự nằm trên các cạnh $B_{1}C_{1},C_{1}A_{1},A_{1}B_{1} $ sao cho
$\angle BAC=\angle B_{1}A_{1}C_{1},
\angle ACB=\angle A_{1}C_{1}B_{1},
\angle CBA=\angle C_{1}B_{1}A_{1} $. Chứng minh rằng trực tâm của hai tam giác ABC và $A_{1}B_{1}C_{1} $ cách đều tâm đường tròn ngoại tiếp tam giác ABC.
[RIGHT][I][B]Nguồn: MathScope.ORG[/B][/I][/RIGHT]
 
macdangnghi is offline   Trả Lời Với Trích Dẫn
The Following User Says Thank You to macdangnghi For This Useful Post:
n.v.thanh (09-03-2012)
Old 09-03-2012, 10:53 AM   #86
nghiepdu-socap
+Thành Viên+
 
nghiepdu-socap's Avatar
 
Tham gia ngày: Apr 2010
Bài gởi: 193
Thanks: 195
Thanked 129 Times in 72 Posts
Trích:
Nguyên văn bởi macdangnghi View Post
Bài 35: Cho tam giác $A_{1}B_{1}C_{1} $ và các điểm A,B,C theo thứ tự nằm trên các cạnh $B_{1}C_{1},C_{1}A_{1},A_{1}B_{1} $ sao cho
$\angle BAC=\angle B_{1}A_{1}C_{1},
\angle ACB=\angle A_{1}C_{1}B_{1},
\angle CBA=\angle C_{1}B_{1}A_{1} $. Chứng minh rằng trực tâm của hai tam giác ABC và $A_{1}B_{1}C_{1} $ cách đều tâm đường tròn ngoại tiếp tam giác ABC.
Bài này là bài hình vòng 1 đề thi chọn đội tuyển dự thi quốc gia của Hà Tĩnh năm nay. Có thể sử dụng phép vị tự quay
[RIGHT][I][B]Nguồn: MathScope.ORG[/B][/I][/RIGHT]
 
nghiepdu-socap is offline   Trả Lời Với Trích Dẫn
Old 09-03-2012, 11:52 AM   #87
kien10a1
+Thành Viên+
 
kien10a1's Avatar
 
Tham gia ngày: Feb 2011
Đến từ: Vĩnh Yên- Vĩnh Phúc
Bài gởi: 371
Thanks: 43
Thanked 263 Times in 153 Posts
Gửi tin nhắn qua Yahoo chát tới kien10a1
Trích:
Nguyên văn bởi macdangnghi View Post
Bài 35: Cho tam giác $A_{1}B_{1}C_{1} $ và các điểm A,B,C theo thứ tự nằm trên các cạnh $B_{1}C_{1},C_{1}A_{1},A_{1}B_{1} $ sao cho
$\angle BAC=\angle B_{1}A_{1}C_{1},
\angle ACB=\angle A_{1}C_{1}B_{1},
\angle CBA=\angle C_{1}B_{1}A_{1} $. Chứng minh rằng trực tâm của hai tam giác ABC và $A_{1}B_{1}C_{1} $ cách đều tâm đường tròn ngoại tiếp tam giác ABC.
Thực ra là nó cũng đã có trên diễn đàn rồi, phần sau của bài này: [Only registered and activated users can see links. ]
[RIGHT][I][B]Nguồn: MathScope.ORG[/B][/I][/RIGHT]
 
__________________
Quay về với nơi bắt đầu
kien10a1 is offline   Trả Lời Với Trích Dẫn
The Following User Says Thank You to kien10a1 For This Useful Post:
n.v.thanh (09-03-2012)
Old 09-03-2012, 03:31 PM   #88
namdung
Administrator

 
Tham gia ngày: Feb 2009
Đến từ: Tp Hồ Chí Minh
Bài gởi: 1,343
Thanks: 209
Thanked 4,066 Times in 778 Posts
Gửi tin nhắn qua Yahoo chát tới namdung
Trích:
Nguyên văn bởi quykhtn View Post
Bài 27.
Cho $ a,b,c $ là các số thực dương thỏa mãn: $ abc=1 $.
Chứng minh rằng:
$ \frac{1}{a^8+b^3+c}+\frac{1}{b^8+c^3+a}+\frac{1}{c ^8+a^3+b} \le 1 $
Hướng dẫn bài này: Áp dụng CBS ta có

$ (a^8 + b^3 + c)(a^{-\frac{8}{3}} + b^{\frac{7}{3}} + c^{\frac{13}{3}}) \ge (a^{\frac{8}{3}} + b^{\frac{8}{3}} + c^{\frac{8}{3}})^2.
$
Sau đó xử lý tiếp, dùng một số bất đẳng thức quen thuộc.

Ý nghĩa quan trọng của bất đẳng thức trên là ta đã quy đồng được mẫu số và đưa về việc chứng minh một bất đẳng thức đối xứng. Dùng abc = 1 để thuần nhất hóa.
[RIGHT][I][B]Nguồn: MathScope.ORG[/B][/I][/RIGHT]
 

thay đổi nội dung bởi: namdung, 09-03-2012 lúc 03:34 PM
namdung is offline   Trả Lời Với Trích Dẫn
The Following User Says Thank You to namdung For This Useful Post:
n.v.thanh (09-03-2012)
Old 09-03-2012, 08:58 PM   #89
king_math96
+Thành Viên+
 
king_math96's Avatar
 
Tham gia ngày: Feb 2010
Đến từ: huyện lặng gió, tỉnh quan họ
Bài gởi: 170
Thanks: 156
Thanked 87 Times in 50 Posts
Bài 36: Tìm hằng số $k$ tốt nhất để BĐT sau đúng $\forall x_{k}>0(k=\overline{1,n})$ thỏa mãn:$x_1 \le x_2 \le ... \le x_{n}$:
$$k.\frac{\sum\limits_{1 \le i<j \le n}(x_{i}-x_{j})^2}{x_{n}} \le \frac{1}{n}\left(\sum_{k=1}^{n}x_{k} \right)-\sqrt[n]{\prod_{k=1}^{n}x_{k}} \le k.\frac{\sum\limits_{1 \le i<j \le n}(x_{i}-x_{j})^2}{x_1}$$.
Nguồn: Nguyễn Bảo Phúc
[RIGHT][I][B]Nguồn: MathScope.ORG[/B][/I][/RIGHT]
 
__________________
Giang hồ đẫm máu anh không sợ
Chỉ sợ đường về vắng bóng em.
king_math96 is offline   Trả Lời Với Trích Dẫn
Old 10-03-2012, 09:13 AM   #90
macdangnghi
+Thành Viên+
 
Tham gia ngày: Jan 2009
Bài gởi: 11
Thanks: 1
Thanked 15 Times in 7 Posts
Bài 36: Cho p nguyên tố. Chứng minh rằng với mọi n nguyên dương tồn tại đa thức Q(x) với hệ số nguyên sao cho Q(1),Q(2),...,Q(n) phân biệt và là lũy thừa của p.
------------------------------
Bài 37: Cho tứ giác lồi ABCD. E,F theo thứ tự thuộc các cạnh AD,BC sao cho $\frac{AE}{ED}=\frac{BF}{FC} $. Tia FE cắt các tia BA,CD lần lượt tại S và T. Chứng minh rằng các đường tròn ngoại tiếp các tam giác SAE,SBF,TCF và TDE cùng đi qua một điểm.
[RIGHT][I][B]Nguồn: MathScope.ORG[/B][/I][/RIGHT]
 

thay đổi nội dung bởi: macdangnghi, 10-03-2012 lúc 09:19 AM Lý do: Tự động gộp bài
macdangnghi is offline   Trả Lời Với Trích Dẫn
The Following User Says Thank You to macdangnghi For This Useful Post:
5434 (10-03-2012)
Trả lời Gởi Ðề Tài Mới

Bookmarks


Quuyền Hạn Của Bạn
You may not post new threads
You may not post replies
You may not post attachments
You may not edit your posts

BB code is Mở
Smilies đang Mở
[IMG] đang Mở
HTML đang Tắt

Chuyển đến


Múi giờ GMT. Hiện tại là 12:35 PM.


Powered by: vBulletin Copyright ©2000-2024, Jelsoft Enterprises Ltd.
Inactive Reminders By mathscope.org
[page compression: 111.99 k/128.57 k (12.90%)]